0% found this document useful (0 votes)
123 views39 pages

Mathematical Reflections _1_2025_Solutions

The document presents several mathematical problems and their solutions, focusing on proving various inequalities and identities involving real numbers. It includes detailed proofs for problems related to algebraic expressions, triangle inequalities, and properties of altitudes in triangles. Multiple contributors from different institutions provide solutions, showcasing a collaborative effort in mathematical problem-solving.

Uploaded by

Florin Rotaru
Copyright
© © All Rights Reserved
We take content rights seriously. If you suspect this is your content, claim it here.
Available Formats
Download as PDF, TXT or read online on Scribd
0% found this document useful (0 votes)
123 views39 pages

Mathematical Reflections _1_2025_Solutions

The document presents several mathematical problems and their solutions, focusing on proving various inequalities and identities involving real numbers. It includes detailed proofs for problems related to algebraic expressions, triangle inequalities, and properties of altitudes in triangles. Multiple contributors from different institutions provide solutions, showcasing a collaborative effort in mathematical problem-solving.

Uploaded by

Florin Rotaru
Copyright
© © All Rights Reserved
We take content rights seriously. If you suspect this is your content, claim it here.
Available Formats
Download as PDF, TXT or read online on Scribd
You are on page 1/ 39

Junior problems

J685. Let a be a nonzero real number. Prove that


 
1 1 1
a3 − + a2
+ − 3 a − =3
a3 a2 a

if and only if    
5 1 3 1 1
a − 5 − 5 a − 3 + 11 a − = 1.
a a a

Proposed by Adrian Andreescu, University of Texas at Dallas, USA

Solution by Daniel Pascuas, Barcelona, Spain


1
Let x = a − . Then
a
     
2 2 1 3 3 1 1 5 5 1 3 1 1
x = a + 2 − 2, x = a − 3 − 3 a − and x = a − 5 − 5 a − 3 + 10 a − ,
a a a a a a
so  
3 1 2 1 1
a − 3 +a + 2 −3 a− = x3 + x2 + 2,
a a a
and    
1 1 1
a5 − − 5 a3
− + 11 a − = x5 + x.
a5 a3 a
Therefore  
3 1 2 1 1
a − 3 +a + 2 −3 a− = 3 ⇐⇒ x3 + x2 − 1 = 0
a a a
and    
5 1 3 1 1
a − 5 − 5 a − 3 + 11 a − = 1 ⇐⇒ x5 + x − 1 = 0.
a a a
Since x5 + x − 1 = (x2 − x + 1)(x3 + x2 − 1) = (x − 12 )2 + 34 (x3 + x2 − 1) and x ∈ R, it is clear that


x5 + x − 1 = 0 is equivalent to x3 + x2 − 1 = 0. Hence the two identities in the statement are equivalent.

Also solved by Arkady Alt, San Jose, California, USA; Brian Bradie, Christopher Newport University,
Newport News, VA, USA; G. C. Greubel, Newport News, VA, USA; Ivan Hadinata, Department of Math-
ematics, Gadjah Mada University, Indonesia; Kousik Sett, India; Nicuşor Zlota, “Traian Vuia” Technical
College, Focşani, Romania; Paolo Perfetti, Università degli studi di Tor Vergata Roma, Italy; Raja Ok-
tovin Parhasian Damanik, Canberra ACT, Australia; Sundaresh H. R., Shivamogga, Karnataka, India;
Telemachus Baltsavias, Kerameies Junior High School, Kefallonia, Greece; Theo Koupelis, Clark College,
Washington, USA; Aaditya Bhaumik Sahu, AMMOC, India; Anderson Torres, Sao Paulo, Brazil; Nikolai
N. Mnev, AMMOC, Bengaluru, India; Polyahedra, Polk State College, FL, USA; Samhith Kadiyala, North
Carolina Cyber Academy, NC, USA; Srijan Sundar, Oxford, UK; Sumanyu Nandecha, AMMOC, India.

Mathematical Reflections 1 (2025) 1


J686. Let a and b be positive real numbers. Prove that
p p p
x2 − ax + a2 + x2 − bx + b2 ≥ a2 + ab + b2
for all real numbers x. When does equality occur?
Proposed by Mircea Becheanu, Canada

Solution 1 by Raja Oktovin Parhasian Damanik, CanberrapACT, Australia p √


If x ≤ 0, p
let x = −y with yp≥ 0. The inequality 2 2 2 2 2 2
p becomes y +pay + a + y + by + b ≥ a + ab + b .
Clearly, y 2 + ay + a2 + y 2 + by + b2 ≥ y 2 − ay + a2 + y 2 − by + b2 . So, it suffices to prove the
original inequality for positive real number x.
Let P A, P B, P C be rays with P A = a, P B = x, and P C = x such that ∠AP B = ∠BP C = 60◦ and
∠AP C = 120◦ .
By cosine law, we have
AB 2 = P A2 + P B 2 − 2 · P A · P B · cos 60◦ = a2 + x2 − ax
BC 2 = P B 2 + P C 2 − 2 · P B · P C · cos 60◦ = x2 + b2 − bx
CA2 = P A2 + P C 2 − 2 · P A · P C · cos 120◦ = a2 + b2 + ab.
√ √ √
By triangle inequality, we have AB +BC ≥ CA, and hence x2 − ax + a2 + x2 − bx + b2 ≥ a2 + ab + b2 .
The equality occurs when B lies on segment AC, and since P B bisects ∠ABC, we calculate P B using
[AP B] + [BP C] = [AP C], i.e.
1 √ 1 √ 1 √ ab
ax 3 + bx 3 = ab 3 ⇐⇒ x = .
4 4 4 a+b

Solution 2 by Daniel Pascuas, Barcelona, Spain


Let d be the Euclidean distance on R2 , i.e. d (x1 , y1 ), (x2 , y2 ) = (x1 − x2 )2 + (y1 − y2 )2 . Then our
 p

inequality directly follows from the triangle inequality for d:


p p √ √ 
x2 − ax + a2 + x2 − bx + b2 = d ( a2 , a 2 3 ), (x, 0) + d (x, 0), ( 2b , − b 2 3 )

√ √  q
2 2
≥ d ( 2 , 2 ), ( 2 , − 2 ) = (a−b)
a a 3 b b 3
4 + 3(a+b) 4
p
= a2 + ab + b2 .
√ √
The equality holds if and only if the point (x, 0) belongs to the segment determined by ( a2 , a 2 3 ) and ( 2b , − b 2 3 ),
ab ab
which means that x = a+b . Thus the equality only occurs when x = a+b .
Solution 3 by Polyahedra, Polk State College, FL, USA
By squaring both sides we see that the inequality is equivalent to
p
2 (x2 − ax + a2 ) (x2 − bx + b2 ) ≥ ab + (a + b)x − 2x2 .
By the Cauchy-Schwarz inequality,
p p
(2x2 − 2ax + 2a2 ) (2x2 − 2bx + 2b2 ) = [a2 + (a − x)2 + x2 ] [b2 + x2 + (b − x)2 ]
≥ ab + (a − x)x + x(b − x) = ab + (a + b)x − 2x2 .
b x b−x ab
Equality holds if and only if a = a−x = x , which is equivalent to x = a+b .

Also solved by Arkady Alt, San Jose, California, USA; Daniel Văcaru, National Economic College “Maria
Teiuleanu”, Pites, ti, Romania; Ioan Viorel Codreanu, Satulung, Maramures, Romania; Ivan Hadinata, De-
partment of Mathematics, Gadjah Mada University, Indonesia; Kousik Sett, India; Marin Chirciu, Colegiul
National Zinca Golescu Piteşti, Romania; Nicuşor Zlota, “Traian Vuia” Technical College, Focşani, Ro-
mania; Paolo Perfetti, Università degli studi di Tor Vergata Roma, Italy; Sundaresh H. R., Shivamogga,
Karnataka, India; Theo Koupelis, Clark College, Washington, USA; Aaditya Bhaumik Sahu, AMMOC, In-
dia; Anderson Torres, Sao Paulo, Brazil; Nihad Hashimov, AMMOC, Azerbaijan; Srijan Sundar, Oxford,
UK; Sumanyu Nandecha, AMMOC, India.

Mathematical Reflections 1 (2025) 2


J687. Let a, b, c be real numbers such that ab + bc + ca = 169 and

(a + 1)3 + (b + 1)3 + (c + 1)3 − 3abc = 1.

Find all possible values of a + b + c.

Proposed by Titu Andreescu, University of Texas at Dallas, USA

Solution by Ivan Hadinata, Department of Mathematics, Gadjah Mada University, Indonesia


Let x = a + b + c and y = ab + bc + ca = 169. Then

0 = (a + 1)3 + (b + 1)3 + (c + 1)3 − 3abc − 1


= (a3 + b3 + c3 − 3abc) + 3(a2 + b2 + c2 ) + 3(a + b + c) + 2
= x(x2 − 3y) + 3(x2 − 2y) + 3x + 2
= x3 + 3x2 − 504x − 1012
= (x + 2)(x + 23)(x − 22),

hence x = −2 or x = −23 or x = 22. Since a, b, c ∈ R, then x2 ≥ 3y = 507, and the only possibility is
x = −23. One example of (a, b, c) when x = −23 is
√ √
a = −8 + 7, b = −8 − 7, c = −7.

In conclusion, the only possible value of a + b + c is −23.

Editor’s Comment. Many readers failed to check the condition (a + b + c)2 ≥ 3(ab + bc + ca).
Raja Oktovin Parhasian Damanik found another triple (a, b, c) satisfying the conditions, arguing as follows.
Take a = −x, b = −y, c = −z, then x + y + z = 23 and√xy + yz + zx = 169. Take z = 5,√then x + y =√18
and xy = 79. This has solution, for example, x, y = 9 ± 2. Hence we can take a = −9 − 2, b = −9 + 2,
and c = −5.
Theo Koupelis noticed that there are infinitely many solutions that satisfy the given equations, arguing as
follows.
We have the system b + c = −23 − a and bc = 169 − a(b + c) = a2 + 23a + 169; thus, b, c are the solutions
of the quadratic t2 + (23 + a)t + (a2 + 23a + 169) = 0, whose
√ discriminant is √ ∆ = −(3a2 + 46a + 147). For
−23 − 2 22 −23 + 2 22
real solutions we must have ∆ ≥ 0; thus, for any ≤a≤ , the values of b and c are
p 3 3
−23 − a ± −(3a2 + 46a + 147)
given by .
2
Also solved by Arkady Alt, San Jose, California, USA; G. C. Greubel, Newport News, VA, USA; Ioan
Viorel Codreanu, Satulung, Maramures, Romania; Kalkin Gandhi, AMMOC Bengaluru, India; Marin Chir-
ciu, Colegiul National Zinca Golescu Piteşti, Romania; Nicuşor Zlota, “Traian Vuia” Technical College,
Focşani, Romania; Paolo Perfetti, Università degli studi di Tor Vergata Roma, Italy; Raja Oktovin Parhasian
Damanik, Canberra ACT, Australia; Sundaresh H. R., Shivamogga, Karnataka, India; Telemachus Balt-
savias, Kerameies Junior High School, Kefallonia, Greece; Theo Koupelis, Clark College, Washington, USA;
Vishwesh Ravi Shrimali, Jaipur, Rajasthan, India; Aaditya Bhaumik Sahu, AMMOC, India; Daniel Pas-
cuas, Barcelona, Spain; Diar Gashi, University of Primorska, Slovenia; Madhav R Pillai, Delhi Public
School, Ranchi, Jharkhand, India; Medha Shrivastava, AMMOC,India; Nihad Hashimov, AMMOC, Azer-
baijan; Nikolai N. Mnev, AMMOC, Bengaluru, India; Polyahedra, Polk State College, FL, USA; Samhith
Kadiyala, North Carolina Cyber Academy, NC, USA; Srijan Sundar, Oxford, UK.

Mathematical Reflections 1 (2025) 3


J688. Prove that positive real numbers x, y, z are the lengths of the altitudes in a triangle if and only if
 
1 1 1 1 1 1
+ + > 2 max , , .
x y z x y z
Proposed by Dan-Ştefan Marinescu, Romania
Solution 1 by Raja Oktovin Parhasian Damanik, Canberra ACT, Australia
Suppose that a triangle has altitudes of lengths x, y, z which are respectively perpendicular to the sides
of length a, b, c. The area [ABC] satisfies 2[ABC] = ax = by = cz. By triangle inequality, we have that
a + b + c > 2max{a, b, c}. Hence,
 
[ABC] [ABC] [ABC] [ABC] [ABC] [ABC]
+ + > 2 max , ,
x y z x y z
 
1 1 1
= 2[ABC] max , ,
x y z
Hence,
 
1 1 1 1 1 1
+ + > 2 max , , .
x y z x y z
1 1 1
Now let x, y, z satisfy the inequality and hence , , are sides of a triangle; let A be the area of that
x y z
1 1 1
triangle. There exists a triangle whose sides are of length , , . The area of this triangle is
2Ax 2Ay 2Az
1
1 1 1 2·
·A = . Hence, the length of the altitude to side of the length will be 4A = x. Similarly,
4A2 4A 2Ax 1
2Ax
the other two altitudes will have length y and z, as desired.

Solution 2 by Daniel Pascuas, Barcelona, Spain


Let ABC be a triangle and, as usual, let a, b, c be the lengths of the sides BC, AC, AB, respectively. Let
x, y, z be the lengths of the altitudes of ABC corresponding to the sides BC, AC, AB, respectively. Then
xa = yb = zc = 2S, where S is the area of the triangle ABC. Therefore a = 2S 2S 2S
x , b = y , and c = z must
satisfy that a + b + c > 2 max{a, b, c}, which means that
 
1 1 1 1 1 1
+ + > 2 max , , .
x y z x y z
Conversely, assume that x, y, z are positive real numbers satisfying this inequality. Then
    
1 1 1 1 1 1 1 1 1 1 1 1
Λ := + + + − + − + − > 0.
x y z y z x x z y x y z
Thus a = x√2 Λ , b = y√2 Λ , c = z √2 Λ are positive real numbers which satisfy a + b + c > 2 max{a, b, c}, and so
they are the lengths of the sides BC, AC, AB, respectively, of a triangle ABC. The semiperimeter of this
triangle is p = 21 (a + b + c) = √1Λ ( x1 + y1 + z1 ), so Heron’s formula shows that its area is equal to
s    
p 1 1 1 1 1 1 1 1 1 1 1 1 1 1
p(p − a)(p − b)(p − c) = + + + − + − + − =√ .
Λ x y z y z x x z y x y z Λ
2 2 2
Hence the lengths of the altitudes corresponding to the sides BC, AC, AB are √
a Λ
= x, √
b Λ
= y, √
c Λ
= z,
respectively.

Also solved by Nicuşor Zlota, “Traian Vuia” Technical College, Focşani, Romania; Theo Koupelis, Clark
College, Washington, USA; Anderson Torres, Sao Paulo, Brazil; Diar Gashi, University of Primorska,
Slovenia; Medha Shrivastava, AMMOC,India; Polyahedra, Polk State College, FL, USA; Srijan Sundar,
Oxford, UK.

Mathematical Reflections 1 (2025) 4


J689. Find the least value of the function
2 1 2
f (x) = + 2 − 2 ,
x2 − 4x + 8 x − 6x + 10 x − 5x + 8
where x is any real number.

Proposed by An Zhenping, Xianyang Normal University, China

Solution 1 by Raja Oktovin Parhasian Damanik, Canberra ACT, Australia


We claim that f (x) ≥ 0 for all real numbers x.
We want to prove that
2 1 2
+ ≥ 2 .
x2 − 4x + 8 x2 − 6x + 10 x − 5x + 8
2 1 2
Note that if x = 2, then LHS = + = 1 and RHS = = 1, so the equality can occur.
4 2 2
To prove the inequality, we use AM-HM inequality:
1 1 4
1 2 + 2 ≥ 3 2 .
2x − 2x + 4 x − 6x + 10 2 x − 8x + 14

3 2 1
It suffices to prove that 2x2 − 10x + 16 ≥ x − 8x + 14. Rearranging, we have x2 − 2x + 2 ≥ 0, i.e.
2 2
x2 − 4x + 4 ≥ 0, i.e. (x − 2)2 ≥ 0 which is true.
Hence, the least value of f (x) is 0, and it is attained by x = 2.

Solution 2 by Daniel Pascuas, Barcelona, Spain


Just note that
1 2x
f (x) = − 2
x2 − 6x + 10 (x − 4x + 8)(x2 − 5x + 8)
(x2 − 4x + 8)(x2 − 5x + 8) − 2x(x2 − 6x + 10)
=
(x2 − 4x + 8)(x2 − 5x + 8)(x2 − 6x + 10)
x4 − 11x3 + 48x2 − 92x + 64
=
(x2 − 4x + 8)(x2 − 5x + 8)(x2 − 6x + 10)
(x − 2)2 (x2 − 7x + 16)
=
(x2 − 4x + 8)(x2 − 5x + 8)(x2 − 6x + 10)
(x − 2)2 (x − 72 )2 + 15

4
= ≥ 0 = f (2), for every x ∈ R,
(x − 2)2 + 4 (x − 52 )2 + 74 ((x − 3)2 + 1)
 

so the the least value of f (x) is 0.

Also solved by Arkady Alt, San Jose, California, USA; Henry Ricardo, Westchester Area Math Circle, New
York, USA; Kalkin Gandhi, AMMOC Bengaluru, India; Nicuşor Zlota, “Traian Vuia” Technical College,
Focşani, Romania; Paolo Perfetti, Università degli studi di Tor Vergata Roma, Italy; Sundaresh H. R.,
Shivamogga, Karnataka, India; Theo Koupelis, Clark College, Washington, USA; Aaditya Bhaumik Sahu,
AMMOC, India; Karma Pillai; Polyahedra, Polk State College, FL, USA; Sumanyu Nandecha, AMMOC,
India.

Mathematical Reflections 1 (2025) 5


J690. Let ABC be a triangle. The angle bisector of ∠CAB intersects BC in D and the circumcircle in X.
Let Y be the orthogonal projection of C on AX and Z the orthogonal projection of D on AB. The
line Y Z intersects BC in E. Prove that AE is perpendicular to BC.

Proposed by Mihaela Berindeanu, Bucharest, Romania

Solution 1 by Raja Oktovin Parhasian Damanik, Canberra ACT, Australia


Let ∠BAC, ∠ABC, ∠BCA are denoted by α, β, γ, respectively.
We want to prove that ∠AEC = ∠AY C = 90◦ , that is, we want to prove that AEY C is a cyclic quadrilateral.
α α
Note that ∠DAE = ∠DAB − ∠BAE = − (90◦ − β) = + β − 90◦ .
2 2 α
Also, note that ∠ECY = ∠DCY = ∠ACY − ∠ACB = 90◦ − − γ.
2
α α
But it is clear now that + β − 90◦ = 90◦ − − γ, i.e. α + β + γ = 180◦ , which is true for angles of a
2 2
triangle α, β, γ.
So, ∠Y AE = ∠DAE = ∠ECY , which implies that AEY C is a cyclic quadrilateral.

Mathematical Reflections 1 (2025) 6


Solution 2 by Diar Gashi, University of Primorska, Slovenia
Let E ′ be the orthogonal projection of A onto BC. Since ∠AE ′ C = 90 = ∠AY C, we have that AE ′ Y C is
cyclic. So,
∠AE ′ Y = 90 + ∠CE ′ Y = 90 + ∠CAY = 90 + ∠BAD.
Since ∠AZD = 90 = ∠AE ′ D, we have that AZE ′ D is cyclic. Thus,

∠AE ′ Z = ∠ADZ = 90 − ∠BAD.

Therefore ∠AE ′ Y + ∠AE ′ Z = 180 and this means that Z, E ′ and Y are collinear.
So, E = E ′ and therefore AE is indeed perpendicular to BC.

Mathematical Reflections 1 (2025) 7


Solution 3 by Kousik Sett, India
Let the circumcircle of triangle ACY intersect BC at E ′ . Then ∠AE ′ C = ∠AY C = 90◦ .
Also, ∠E ′ Y A = ∠E ′ CA = ∠BCA = ∠BXA, which implies E ′ Y ∥ BX.
Since ∠DE ′ A = ∠DZA = 90◦ , quadrilateral AZE ′ D is cyclic. So, ∠BE ′ Z = ∠ZAD = ∠BAX.

Since the angle bisector of ∠CAB intersects the circumcircle of triangle ABC at X, we infer that X is the
midpoint of arc BC not containing A. So, ∠XBC = ∠BCX = ∠BAX.
Therefore, we obtain ∠BE ′ Z = ∠XBC, which implies ZE ′ ∥ BX.
Since E ′ Y ∥ BX and ZE ′ ∥ BX, it follows that Z, E ′ , and Y are collinear, and E ′ = BC ∩ ZY . However,
it is given that E = BC ∩ ZY . Therefore, we conclude that E ′ = E and consequently, we have AE is
perpendicular to BC.

Also solved by Theo Koupelis, Clark College, Washington, USA; Karma Pillai; Polyahedra, Polk State
College, FL, USA; Prasanna Pawar, AMMOC, Bengaluru, India.

Mathematical Reflections 1 (2025) 8


Senior problems

S685. Let a, b be positive real numbers. Prove that


a b a+b
2
+ 2
≥ .
(1 + b) (1 + a) 2(1 + ab)

Proposed by An Zhenping, Xianyang Normal University, China

Solution 1 by Arkady Alt, San Jose, California, USA


By Cauchy-Schwarz Inequality, we have
 
b a 
(1 + b)2 ≤ + 1 (ab + 1) , (1 + a)2 ≤ + 1 (ab + 1) .
a b

Then
a b a b
2 + ≥ + a
(1 + a)2 b
 
(1 + b) a + 1 (ab + 1) b + 1 (ab + 1)
 2 2

1 a b
= +
ab + 1 a + b a + b
1
· a2 + b2

=
(ab + 1) (a + b)
1 (a + b)2
≥ ·
(ab + 1) (a + b) 2
a+b
= .
2 (1 + ab)

The equality holds if and only if a = b = 1.

Solution 2 by Ivan Hadinata, Department of Mathematics, Gadjah Mada University, Indonesia


By the AM-GM Inequality, a + b + a2 b + ab2 ≥ 4ab. Hence

2(a + b)(1 + ab) ≥ a + b + 4ab + a2 b + ab2 = a(1 + b)2 + b(1 + a)2 .

Then, by Cauchy-Schwarz Inequality we have

a b a2 b2 (a + b)2 (a + b)2 a+b


2
+ 2
= 2
+ 2
≥ 2 2
≥ = ,
(1 + b) (1 + a) a(1 + b) b(1 + a) a(1 + b) + b(1 + a) 2(a + b)(1 + ab) 2(1 + ab)

as wanted. The equality holds if and only if a = b = 1.

Also solved by Daniel Văcaru, National Economic College “Maria Teiuleanu”, Pites, ti, Romania; G. C.
Greubel, Newport News, VA, USA; Henry Ricardo, Westchester Area Math Circle, New York, USA; Ioan
Viorel Codreanu, Satulung, Maramures, Romania; Marin Chirciu, Colegiul National Zinca Golescu Piteşti,
Romania; Miguel Amengual Covas, Cala Figuera, Mallorca, Spain; Nicuşor Zlota, “Traian Vuia” Tech-
nical College, Focşani, Romania; Paolo Perfetti, Università degli studi di Tor Vergata Roma, Italy; Raja
Oktovin Parhasian Damanik, Canberra ACT, Australia; Sundaresh H. R., Shivamogga, Karnataka, India;
Theo Koupelis, Clark College, Washington, USA; Aaditya Bhaumik Sahu, AMMOC, India; Daniel Pascuas,
Barcelona, Spain; Diar Gashi, University of Primorska, Slovenia; Nihad Hashimov, AMMOC, Azerbaijan;
Prasanna Pawar, AMMOC, Bengaluru, India; Srijan Sundar, Oxford, UK; Sumanyu Nandecha, AMMOC,
India.

Mathematical Reflections 1 (2025) 9


S686. Let a, b, c be positive real numbers such that a + b + c = 3. Prove that

1 1 1 3 2 1
√ +√ +√ ≤ +√ .
2
b +c 2 2
c +a 2 2
a +b 2 ab + bc + ca 2

Proposed by Marius Stănean, Zalău, Romania

Solution by the author


By homogenizing the inequality becomes
1 1 1 a+b+c 3
p +p +p ≤ + .
2 2
2(b + c ) 2 2
2(c + a ) 2 2
2(a + b ) ab + bc + ca 2(a + b + c)

We have
X 1 X 1 (a + b + c)2 + ab + bc + ca
p ≤ = .
cyc 2(b2 + c2 ) cyc
b+c (a + b + c)(ab + bc + ca) − abc

Hence it suffices to prove that


(a + b + c)2 + ab + bc + ca a+b+c 3
≤ + .
(a + b + c)(ab + bc + ca) − abc ab + bc + ca 2(a + b + c)
Clearing denominators and expanding, the inequality becomes

(a + b + c)(ab + bc + ca)2 ≥ 2abc(a + b + c)2 + 3abc(ab + bc + ca).

But this follows from the AM-GM Inequality and (x + y + z)2 ≥ 3(xy + yz + zx). Indeed, we have

2(a + b + c)(ab + bc + ca)2 2(a + b + c)


= · (ab + bc + ca)2
3 3
2(a + b + c)
≥ · 3abc(a + b + c) = 2abc(a + b + c)2
3
and
(a + b + c)(ab + bc + ca)2 (a + b + c)(ab + bc + ca)
= · (ab + bc + ca)
3 3
≥3abc(ab + bc + ca).

Summing up these inequalities, we get the desired result.


Observation. This is stronger than problem 1 from 2024 Junior Balkan MO:
Let a, b, c be positive real numbers such that
1
a2 + b2 + c2 = .
4
Prove that √
1 1 1 2
√ +√ +√ ≤ .
2
b +c 2 2
c +a 2 2
a +b2 (a + b)(b + c)(c + a)
It can be shown, not hard, like
a+b+c 3 3(a + b + c) 4(a2 + b2 + c2 )
+ ≤ ≤ .
ab + bc + ca 2(a + b + c) 2(ab + bc + ca) (a + b)(b + c)(c + a)

Also solved by Arkady Alt, San Jose, California, USA; Nicuşor Zlota, “Traian Vuia” Technical College,
Focşani, Romania; Paolo Perfetti, Università degli studi di Tor Vergata Roma, Italy; Theo Koupelis, Clark
College, Washington, USA; Aaditya Bhaumik Sahu, AMMOC, India; Nguyen Thanh Nam, Tu Son Sec-
ondary School, Vietnam.

Mathematical Reflections 1 (2025) 10


S687. Solve in real numbers the system of equations:
s r r
x2 + 1 y2 + 1 z2 + 1
= x, 5 = 6y, 7 = 20z.
y2 + 1 z2 + 1 x2 + 1

Proposed by Titu Andreescu, University of Texas at Dallas, USA

Solution 1 by the author  π


Because x, y, z are positive, let x = tan u, y = tan v, z = tan w, where u, v, w ∈ 0, . Then
2
cos v sin u 5 cos w 6 sin v 7 cos u 20 sin w
= , = , = ,
cos u cos u cos v cos v cos w cos w
implying
cos v = sin u, 5 cos w = 6 sin v, 7 cos u = 20 sin w.
It follows that sin v = cos u and so 4(5 cos w) = 4(6 cos u) and, since 20 sin w = 7 cos u, we get

202 (cos2 w + sin2 w) = (242 + 72 ) cos2 u,


4 3 7 24 4 3
yielding cos u = . Then sin u = , sin w = , cos w = , sin v = , cos v = , hence
5 5 25 25 5 5
3 4 7
x = tan u = , y = tan v = , z = tan w = .
4 3 24

Solution 2 by Theo Koupelis, Clark College, Washington, USA


Clearly we have x, y, z > 0. Multiplying the three given equations we get 35 = 120xyz or

24xyz = 7. (1)

Squaring and simplifying the first of the given equations we get x2 + 1 = x2 (y 2 + 1) or

xy = 1. (2)

From (1) and (2) we get z = 7/24. Substituting into the second of the given equations, squaring and
simplifying we get
y2 + 1 16 4
25 · 2
= 36y 2 =⇒ y 2 = =⇒ y = .
(7/24) + 1 9 3
 
3 4 7
Therefore, (x, y, z) = , , is the only solution.
4 3 24

Also solved by Arkady Alt, San Jose, California, USA; G. C. Greubel, Newport News, VA, USA; Marin
Chirciu, Colegiul National Zinca Golescu Piteşti, Romania; Nicuşor Zlota, “Traian Vuia” Technical College,
Focşani, Romania; Paolo Perfetti, Università degli studi di Tor Vergata Roma, Italy; Raja Oktovin Parhasian
Damanik, Canberra ACT, Australia; Sundaresh H. R., Shivamogga, Karnataka, India; Daniel Pascuas,
Barcelona, Spain; Diar Gashi, University of Primorska, Slovenia; Madhav R Pillai, Delhi Public School,
Ranchi, Jharkhand, India; Nihad Hashimov, AMMOC, Azerbaijan; Nikolai N. Mnev, AMMOC, Bengaluru,
India; Srijan Sundar, Oxford, UK.

Mathematical Reflections 1 (2025) 11


S688. Prove that
ab2 + a + b2
a2 + ab + b
is an integer for infinitely many positive integers a and infinitely many positive integers b.

Proposed by Mircea Becheanu, Canada

Solution 1 by Theo Koupelis, Clark College, Washington, USA


For any positive integer a we set b = a[a(a − 1) + 1] = a3 − a2 + a, which is clearly a positive integer. Then

ab2 + a + b2 (a + 1)(a3 − a2 + a)2 + a a(a3 + a + 1)(a3 − a2 + 1)


= = = a3 − a2 + 1,
a2 + ab + b (a + 1)(a3 − a2 + a) + a2 a(a3 + a + 1)

which is clearly a positive integer.

Solution 2 by Daniel Pascuas, Barcelona, Spain


Just note that ab2 + a + b2 = (a2 + ab + b)(b − a + 1) + a3 − a2 + a − b so

ab2 + a + b2
= b − a + 1 is an integer if b = p(a), where p(x) = x3 − x2 + x.
a2 + ab + b
Since p(a) is a positive integer, for any positive integer a, and p(x) is increasing on R (because p′ (x) =
3x2 − 2x + 1 = 3(x2 − 32 x + 13 ) = 3(x − 13 )2 + 23 > 0, for any x ∈ R), the values of p(x) on the positive integers
form an infinite set of positive integers. Therefore we conclude that

ab2 + a + b2
a2 + ab + b
is an integer for infinitely many positive integers a and infinitely many positive integers b.

Also solved by Anderson Torres, Sao Paulo, Brazil.

Mathematical Reflections 1 (2025) 12


S689. Let ABC be an acute triangle with incenter I, A1 , B1 , C1 the feet of the altitudes and A2 , B2 , C2 the
midpoints of the sides BC, AC, AB, respectively. The line A2 I intersects AA1 in A3 , the line B2 I
−−→ −−→ −−→ → −
intersects BB1 in B3 and the line C2 I intersects CC1 in C3 . Prove that if AA3 + BB3 + CC3 = 0
then ABC is equilateral.

Proposed by Mihaela Berindeanu, Bucharest, Romania

Solution by Kousik Sett, India


Let the altitudes AA1 , BB1 , and CC1 meet at H (the orthocenter of triangle ABC). Since A3 ∈ AA1 ,
−−→ −−→ −−→ → − −−→ −−→ −−→ → −
B3 ∈ BB1 , C3 ∈ CC1 , and AA3 + BB3 + CC3 = 0 , we infer that AH + BH + CH = 0 .



Let the position vectors of the vertices A, B, C be → −
α , β , and →

γ respectively. From right triangles AA1 B
and AA1 C, we have
tan C AA1 BA1 BA1
= · = .
tan B A1 C AA1 A1 C
Consequently, if the position vector of A is →
1

a , then
1




− (tan B) β + (tan C)→

γ
a1 = . (1)
tan B + tan C
Also, we have
tan C BA1 BA1 BA1 a tan C
= = = ⇒ BA1 = .
tan B + tan C BA1 + A1 C BC a tan B + tan C
Again, since ∠CBB1 = 90◦ − C and ∠B1 BA = 90◦ − A, we have
1
A1 H [A1 BH] 2 · BA1 · BH · sin CBB1 BA1 · cos C
= = 1 = .
HA [HBA] 2 · BH · BA · sin B1 BA c · cos A

Substituting the expressions for BA1 , and using the Sine law, we obtain
A1 H a tan C cos C sin A · sin C 1 tan A
= · = · = .
HA tan B + tan C c · cos A tan B + tan C sin C · cos A tan B + tan C


Consequently, if the position vector of H is h , then using (3), we obtain



− (tan A)→
−α + (tan B + tan C)→

a1 (tan A)→
−α + (tan B) β + (tan C)→

γ
h = = .
tan A + tan B + tan C tan A + tan B + tan C

Mathematical Reflections 1 (2025) 13


−−→ −−→ −−→ → − →
− − →
− → − →
− − →

Since AH + BH + CH = 0 , we have ( h − →
α)+(h − β )+(h −→
γ ) = 0 , which implies

− − →
− →

(tan B)( β − →
α ) + (tan C)(→

γ −→

α ) + (tan A)(→

α − β ) + (tan C)(→−
γ − β)

− − →

+ (tan A)(→

α −→−
γ ) + (tan B)( β − →
γ)= 0,

which yields

− →

(2 tan A − tan B − tan C)→

α + (2 tan B − tan C − tan A) β + (2 tan C − tan A − tan B)→

γ = 0.


Since →

α , β , and →

γ are non-collinear vector, we must have

2 tan A − tan B − tan C = 0, 2 tan B − tan C − tan A = 0, and 2 tan C − tan A − tan B = 0.

Therefore,
3 tan A = tan A + tan B + tan C = 3 tan B = 3 tan C,
and since A, B, and C are acute angles, we conclude that A = B = C. Hence the triangle must be
equilateral.

Also solved by Theo Koupelis, Clark College, Washington, USA.

Mathematical Reflections 1 (2025) 14


S690. Let
2a 2b 2c
x= , y= , z= ,
b+c c+a a+b
where a, b, c are positive real numbers such that at most one of them is less that 1 and abc = 1. Prove
that
1 1 1
+ + + 3 ≥ 2(x + y + z).
x y z

Proposed by Vasile Cı̂rtoaje, Oil-Gas University, Ploieşti, Romania

Solution by the author


Assume that a ≥ b ≥ 1 ≥ c and denote p = a + b + c and q = ab + bc + ca. By the AM-GM inequality, we
have
p ≥ 3(abc)1/3 = 3.
In addition, from (a − 1)(b − 1)(c − 1) ≤ 0, we get

q ≥ p.

Write the inequality as follows:


Xb+c X 8a
+6≥ ,
cyc
a cyc
b+c
X 8 X
bc(b + c) + 6 ≥ a(a + b)(a + c),
cyc
(b + c)(c + a)(a + b) cyc

8(p3 − 2pq + 3)
pq + 3 ≥ .
pq − 1
So, for fixed p, we need to show that f (q) ≥ 0, where

f (q) = p2 q 2 + 18pq − 8p3 − 27.

Since f (q) is increasing, we have

f (q) ≥ f (p) = p4 − 8p3 + 18p2 − 27 = (p + 1)(p3 − 9p2 + 27p − 27) = (p + 1)(p − 3)3 ≥ 0.

The equality occurs for a = b = c = 1.

Also solved by Arkady Alt, San Jose, California, USA; Marin Chirciu, Colegiul National Zinca Golescu
Piteşti, Romania; Nicuşor Zlota, “Traian Vuia” Technical College, Focşani, Romania; Paolo Perfetti, Uni-
versità degli studi di Tor Vergata Roma, Italy; Theo Koupelis, Clark College, Washington, USA.

Mathematical Reflections 1 (2025) 15


Undergraduate problems

U685. Let {an } be the sequence defined by a1 = 2 and an = 2a2n−1 − 1, n = 2, 3, . . . . Evaluate

2n−1 (an−1 + 1)(an−2 + 1) · · · (a1 + 1)


lim .
n→∞ an

Proposed by Nguyen Viet Hung, Hanoi University of Science, Vietnam

Solution by 1 the author


From an = 2a2n−1 − 1 we obtain

an − 1 = 2(a2n−1 − 1) = 2(an−1 − 1)(an−1 + 1).

This gives us
an − 1
= 2(an−1 + 1).
an−1 − 1
Similarly
an−1 − 1
= 2(an−2 + 1),
an−2 − 1
... ... ...,
a2 − 1
= 2(a1 + 1).
a1 − 1
Multiplying these relations we get
an − 1
2n−1 (an−1 + 1)(an−2 + 1) · · · (a1 + 1) = = an − 1.
a1 − 1
Therefore
2n−1 (an−1 + 1)(an−2 + 1) · · · (a1 + 1) 1
=1− .
an an
It’s easy to check that an > 1 forall n. Furthermore

an − an−1 = 2a2n−1 − an−1 − 1 = (an−1 − 1)(2an−1 + 1) > 0 ∀ n.

Hence {an } is a increasing sequence. Now if {an } has a finite upper bound, then there exits ℓ = lim an
n→∞
where ℓ is finite. Since an > a1 = 2 forall n then ℓ ≥ 2. From an = 2a2n−1 − 1 we take n → ∞ to get
ℓ = 2ℓ2 − 1. This yields (ℓ − 1)(2ℓ + 1) = 0 or ℓ = 1 which is a contradiction. So lim an = ∞. From here
n→∞
we obtain
2n−1 (an−1 + 1)(an−2 + 1) · · · (a1 + 1)
 
1
lim = lim 1 − = 1.
n→∞ an n→∞ an

Solution by 2 Seán M. Stewart, King Abdullah University of Science and Technology, Saudi Arabia
Denote the limit to be found by L. We show that L = 1.
We first show that an → ∞ as n → ∞. To do this, an explicit expression for an will first be found, before
taking the appropriate limit. Let  
1 1
a1 = x+ , x > 1.
2 x

Since a1 = 2, on solving for x, for x > 1 we find x = 2 + 3. Next

1 2
    
1 1 1
a2 = 2a21 − 1 = 2 x+ −1= x2 + 2 .
2 x 2 x

Mathematical Reflections 1 (2025) 16


By induction on n, it can be readily shown that
 
1 2n−1 1
an = x + 2n−1 ,
2 x

where n is a positive integer and x = 2 + 3. From this explicit expression we readily see that

an → ∞ as n → ∞.

Now consider the limit. Using the given recurrence relation for the sequence {an }n≥1 , namely a1 = 2 and

an = 2a2n−1 − 1, n = 2, 3, . . .

allows the limit to be rewritten as


2n−1 (an−1 + 1)(an−2 + 1) · · · (a1 + 1)
L = lim
n→∞ an
2n−1 · 2an−2 · 2an−3 · · · 2a21 · 3
2 2
= lim
n→∞ an
2n−1 12 (2an−2 )2 · 12 (2an−3 )2 · 12 (2a1 )2 · 3
= lim
n→∞ an
2n−1 (2an−2 ) (2an−3 )2 · · · (2a1 )2 · 3
2
= lim
n→∞ 2n−2 an
n−2
1 Y
= 6 lim (2ak )2 .
n→∞ an
k=1

Now
2(a2k+1 − 1) 2(a2k+1 − 1) a2k+1 − 1
(2ak )2 = 2(2a2k ) = 2(ak+1 + 1) = = = .
ak+1 − 1 (2a2k − 1) − 1 a2k − 1
Returning to the limit and making use of the above result we have
n−2
1 Y a2k+1 − 1
L = 6 lim
n→∞ an a2k − 1
k=1
1 a2n−1 − 1
= 6 lim · 2
n→∞ an a1 − 1
a 2 −1
= 2 lim n−1
n→∞ an
a 2 −1
= 2 lim n−1
n→∞ 2a2
n−1 − 1
1
1− a2n−1
= 2 lim 1
n→∞ 2− a2n−1
1
=2· = 1,
2
since an → ∞ as n → ∞, as announced.

Solution by 3 Brian
√ Bradie, Christopher Newport University, Newport News, VA, USA
Let x = ln(2 + 3). Then
√ √ √
2 + 3 + 2+1√3 2+ 3+2− 3
cosh x = = = 2,
2 2

Mathematical Reflections 1 (2025) 17


so a1 = cosh x. Next, a2 = 2 cosh2 x − 1 = cosh(2x) and by a straightforward induction argument, an =
cosh(2n−1 x) for all n ≥ 1. From here, we find

j−1 x j−1 x
 j−2 j−2
2
2(aj + 1) = e2 + 2 + e−2 = e2 x + e−2 x

and
n−1
Y 2
j−2 x j−2 x
2n−1 (an−1 + 1)(an−2 + 1) · · · (a1 + 1) = e2 + e−2
j=1
 2
n−1
2X
n−1 −2j+1)x/2
=  e(2 
j=1
n−1 −1)x/2 n−1 −1)x/2
!2
e(2 − e(−2
=
1 − e−x
n−2 n−2
!2
e2 x − e−2 x
=
ex/2 − e−x/2
sinh2 (2n−2 x)
= .
sinh2 (x/2)

Note that √ √ √
1√
2+ 3−2+ 2+ 3 3+2− 3 1
2
sinh (x/2) = = = ,
4 4 2
so
sinh2 (2n−2 x)
= 2 sinh2 (2n−2 x) = 2(cosh2 (2n−2 x) − 1)
sinh2 (x/2)
= (2 cosh2 (2n−2 x) − 1) − 1 = cosh(2n−1 x) − 1.

Finally,
2n−1 (an−1 + 1)(an−2 + 1) · · · (a1 + 1) cosh(2n−1 x) − 1
lim = lim = 1.
n→∞ an n→∞ cosh(2n−1 x)

Also solved by Arkady Alt, San Jose, California, USA; Devis Alvarado, UNAH y UPNFM, Tegucigalpa,
Honduras; G. C. Greubel, Newport News, VA, USA; Joshua Pité, Cambridge Rindge and Latin School,
MA, USA; Juan José Granier, Cencosud, Chile; Nicuşor Zlota, “Traian Vuia” Technical College, Focşani,
Romania; Paolo Perfetti, Università degli studi di Tor Vergata Roma, Italy; Theo Koupelis, Clark College,
Washington, USA; Anderson Torres, Sao Paulo, Brazil; Diar Gashi, University of Primorska, Slovenia.

Mathematical Reflections 1 (2025) 18


U686. Let f : [0, 1] −→ R be a convex function such that f (0) = 0. Prove that
Z 1 2 Z 1
3
f (x)dx ≤ f (x)2 dx.
0 4 0

Proposed by Robert Rogozsan, Baia Mare, Romania

Solution by Daniel Pascuas, Barcelona, Spain


The inequality at the statement is equivalent to
Z 1
√ Z 1 1
3 2
f (x) dx ≤ f (x)2 dx . (1)
0 2 0

We are going to prove that


Z 1
√ Z 1 1
3 2
2
f (x) dx ≤ f (x) dx , (2)
0 2 0
R1
which clearly implies (1) whenever 0 f (x) dx ≥ 0. However we will show that (1) fails for many convex
functions f : [0, 1] → R such that f (0) = 0. By the way, note that the inequality (2) is sharp since it
becomes an equality for f (x) = x.
Let us prove (2). Since f is convex and f (0) = 0, we have that

f (tx) = f tx + (1 − t)0 ≤ tf (x) + (1 − t)f (0) = tf (x) (t, x ∈ [0, 1]),

so Z x Z 1 Z 1
1
f (t) dt = x f (tx) dt ≤ xf (x) t dt = xf (x) (x ∈ [0, 1]).
0 0 0 2
By integrating this inequality and applying Fubini’s theorem, we get that

1 1
Z Z 1 Z x  Z 1 Z 1 
xf (x) dx ≥ f (t) dt dx = f (t) dx dt
2 0 0 0 0 t
Z 1 Z 1 Z 1
= (1 − t)f (t) dt = f (x) dx − xf (x) dx,
0 0 0

and therefore Z 1 Z 1
3
f (x) dx ≤ xf (x) dx.
0 2 0
Note that we may apply Fubini’s theorem because the convexity of f on [0, 1] implies that f is bounded
and continuous on (0, 1) (see, for example, Proposition 1.1.3 in C. P. Niculescu and L.-P. Persson, Convex
Functions and Their Applications - A contemporary approach, Second Edition, CMS Books in Mathematics,
Springer, 2018). Hence Cauchy-Schwarz inequality gives (2):
Z 1 Z 1 Z 1  1 Z 1 1 √ Z 1 1
3 3 2
2
2
2 3 2
2
f (x) dx ≤ xf (x) dx ≤ x dx f (x) dx = f (x) dx .
0 2 0 2 0 0 2 0

Finally, for any c > 21 , consider fc (x) = (x − c)2 − c2 = x2 − 2cx, which is a convex function on [0, 1] and
satisfies fc (0) = 0. Then
Z 1 Z 1 Z 1
1 2 1 4c2
fc (x) dx = − c and fc (x) dx = (x4 − 4cx3 + 4c2 x2 ) dx = − c + ,
0 3 0 0 5 3
so 1 √ Z 1 1
Z 1 
1 3 3c 2 3 2
fc (x) dx = c − > − + c2 = 2
fc (x) dx ,
0 3 20 4 2 0

Mathematical Reflections 1 (2025) 19


because (c − 13 )2 − ( 20
3
− 3c
4 + c2 ) = c
12 − 7
180 = 15c−7
180 > 0, since c > 12 . Hence (1) fails for fc .

Editor’s Comment. Paolo Perfetti provided the following counterexample for f . Let f (x) be the convex
function equal to −xn for 0 ≤ x ≤ 1/n, f (x) = −1 for 1/n ≤ x ≤ 1−1/n, f (x) = (x−1)n for 1−1/n ≤ x ≤ 1.
2
1
1 2 3 1 4 2
Z   Z  
2 3
f (x)dx = 1− ̸≤ (f (x)) dx = 1−
0 n 4 0 4 3n

for any n ≥ 2.

Also solved by Artem Borisov; G. C. Greubel, Newport News, VA, USA; Joshua Pité, Cambridge Rindge and
Latin School, MA, USA; Nicuşor Zlota, “Traian Vuia” Technical College, Focşani, Romania; Paolo Perfetti,
Università degli studi di Tor Vergata Roma, Italy; Theo Koupelis, Clark College, Washington, USA.

Mathematical Reflections 1 (2025) 20


U687. Prove that
1

3
√ Z 1√ √ Z 1 √ √
1+x+ 31−x 3
1+x− 31−x 3
1+x+ 31−x
Z
4
√ dx < √ dx < √ dx.
4 x 0 x 0 x
5

Proposed by Alessandro Ventullo, Milan, Italy

Solution 1 by the author


1 8 4 5 8 1
Since + log 2 < and − log 2 < , it is sufficient to prove that
6 9 5 6 9 4
Z 1 √
3
√ Z 1√ √ Z 5 − 8 log 2 √ √
1+x+ 31−x 3
1+x− 31−x 6 9
3
1+x+ 31−x
√ dx ≤ √ dx ≤ √ dx.
1
+ 8 log 2 x 0 x 0 x
6 9

First, we observe that the last two improper integrals converge because
√3
√ 2
1+x− 31−x 3x
√ ∼√ →0 as x → 0
x x
and √ √
3
1+x+ 31−x 2
√ ∼√
x x
and the integral of the last function converges as x → 0. In order to conclude, it is sufficient to use Steffensen’s
Inequality: if f and g are two integrable functions on [a, b] such that f is decreasing and 0 ≤ g(x) ≤ 1, then
Z b Z b Z a+λ
f (x) dx ≤ f (x)g(x) dx ≤ f (x) dx,
b−λ a a
Z b
where λ = g(x) dx.
a√
3
√ √ √
1+x+ 31−x 3
1+x− 31−x
By taking f (x) = √ and g(x) = √3
√ , it is easy to check that these functions
x 1+x+ 31−x
are integrable on [0, 1], f (x) is decreasing and 0 ≤ g(x) ≤ 1 for all x ∈ [0, 1]. Moreover, from Problem U531
proposed by the author we know that
Z 1√3

1+x− 31−x 5 8
λ= √
3
√ 3
dx = − log 2,
0 1+x+ 1−x 6 9
and the conclusion follows.

Solution 2 by Daniel Pascuas, Barcelona, Spain



By applying the substitution t = x we may write the inequality to be proved as
1
Z 1 p p Z 1 p p Z
2 p p
3 2 3 2
 3 2 3 2
 3 3 
1 + t + 1 − t dt < 1 + t − 1 − t dt < 1 + t2 + 1 − t2 dt. (∗)
√2 0 0
5

Taking into account the binomial expansion



√ 1 X
3
1 + x = (1 + x) 3 = cn xn (−1 < x < 1),
n=0
1 
where cn = 3
n , we get that
∞ ∞
√ √ X √ √ X
3
1+x− 3
1−x=2 c2k+1 x2k+1 and 3
1+x+ 3
1−x=2 c2k x2k ,
k=0 k=0

Mathematical Reflections 1 (2025) 21


for 0 < x < 1, and, in particular,
p p ∞
X ∞
X
3 3
1 + t2 − 1 − t2 = 2 c2k+1 t4k+2 = 2
3 +2 c2k+1 t4k+2 and
k=0 k=1
p p ∞
X ∞
X
3 3
1 + t2 + 1 − t2 = 2 c2k t4k = 2 + 2 c2k t4k , for 0 < t < 1.
k=0 k=1

But cn = 13 ( 31 − 1) · · · ( 13 − (n − 1)) n!
1
, for n ≥ 1, so c2k < 0 < c2k+1 , for k ≥ 1, and therefore
p
3
p
3
p
3
p
3
2
1 + t2 + 1 − t2 < 2 and 3 < 1 + t2 − 1 − t2 , for 0 < t < 1.

Hence we obtain the first inequality in (∗):


Z 1   Z 1 p
p
3 2
p
3 2
 2 2 3
p
3 
1 + t + 1 − t dt < 2 1 − √ < < 1 + t2 − 1 − t2 dt.
2
√ 5 3 0
5

The second inequality in (∗) can be written as


1
Z 1 p p Z
2 p
3 3  3
1 + t2 − 1 − t2 dt < 2 1 − t2 dt,
1
2
0

which holds because


Z 1 p Z 1 q q
2
p 
3 3
1 + (t + 21 )2 − 3 1 − (t + 12 )2 dt
 3
2 2
1 + t − 1 − t dt =
1
2
0

and q q p
3
3
1 + (t + 12 )2 − 3 1 − (t + 12 )2 < 2 1 − t2 (0 < t < 21 ),
√ q q
since the function f (t) = 2 3 1 − t2 + 3 1 − (t + 12 )2 − 3 1 + (t + 12 )2 satisfies that:

• f is decreasing on [0, 21 ]: Just note that


2 − 2 − 2
f ′ (t) = 43 t(1 − t2 )− 3 − 23 (t + 12 ) 1 − (t + 21 )2 3 − 32 (t + 12 ) 1 + (t + 12 )2 3
− 2 − 2
< 13 4t − 2(t + 21 ) 1 − (t + 12 )2 3 = 13 (2t − 1) 1 − (t + 12 )2 3 ≤ 0,


for any t ∈ [0, 21 ].


q √ √ √
• f ( 21 ) = 2 3 34 − 3 2 = 3 6 − 3 2 > 0.

Also solved by Joshua Pité, Cambridge Rindge and Latin School, MA, USA; Paolo Perfetti, Università degli
studi di Tor Vergata Roma, Italy; Theo Koupelis, Clark College, Washington, USA.

Mathematical Reflections 1 (2025) 22


U688. Evaluate n
1
xn ln(1 + x)
 Z
lim 1 + dx .
n→∞ 0 1 + xn

Proposed by Mihaela Berindeanu, Bucharest, Romania

Solution 1 by Seán M. Stewart, King Abdullah University of Science and Technology, Saudi Arabia
Denote the limit to be evaluated by L. We show that
2
L = elog (2)
.

Write the limit as


L = lim (1 + In )n ,
n→∞

where
1
xn log(1 + x)
Z
In = dx.
0 1 + xn
Setting x = t1/n in the integral gives
1 1/n
+ t1/n )
Z
1 t log(1
In = dt.
n 0 1+t

Note that as n → ∞, for t ∈ (0, 1) we see that t1/n → 1. So for n → ∞, by the dominated convergence
theorem we have
1 1 1 · log(1 + 1) log(2) 1 dt
Z Z
1
In → dt = = log2 (2).
n 0 1+t n 0 1+t n
So for the limit, by the dominated convergence theorem we have
n
log2 (2)

2
L = lim 1 + = elog (2) ,
n→∞ n

as announced. Here the well known result of


 x n
ex = lim 1+ ,
x→∞ n
with x = log2 (2) has been used.

Solution 2 by Daniel Pascuas, Barcelona, Spain


First, we integrate by parts to obtain that
Z 1 n
1 1
Z
x ln(1 + x)
n
dx = x ln(1 + x) d ln(1 + xn )
0 1 + x n 0
 1 Z 1 
1 n n

= x ln(1 + x) ln(1 + x ) − ln(1 + x ) d x ln(1 + x)
n 0 0
Z 1 Z 1
x ln(1 + xn )
 
1 2 n
= (ln 2) − ln(1 + x ) ln(1 + x) dx − dx .
n 0 0 1+x

Since 0 ≤ ln(1 + t) ≤ t, for any t ≥ 0, we have that


1 1
x ln(1 + xn )
Z Z
1
0≤ xn+1 dx =
dx ≤ and
0 1+x 0 n + 2
Z 1 Z 1
n ln 2
0≤ ln(1 + x ) ln(1 + x) dx ≤ (ln 2) xn dx = .
0 0 n+1

Mathematical Reflections 1 (2025) 23


It follows that
1
xn ln(1 + x) (log 2)2
Z  
1
dx = +O 2 ,
0 1 + xn n n
so, taking into account that ln(1 + t) = t + O(t2 ), as t → 0, we deduce that
1 n Z 1 n
xn ln(1 + x)
 Z   
x ln(1 + x)
1+ dx = exp n ln 1 + dx
0 1 + xn 0 1 + xn
(ln 2)2
   
1
= exp n ln 1 + +O
n n2
  
1
= exp (ln 2)2 + O .
n

Therefore n
1
xn ln(1 + x)
 Z
2
lim 1 + dx = e(ln 2) = 2ln 2 .
n→∞ 0 1 + xn

Also solved by G. C. Greubel, Newport News, VA, USA; Nicuşor Zlota, “Traian Vuia” Technical College,
Focşani, Romania; Paolo Perfetti, Università degli studi di Tor Vergata Roma, Italy; Theo Koupelis, Clark
College, Washington, USA; Srijan Sundar, Oxford, UK.

Mathematical Reflections 1 (2025) 24


U689. Evaluate the integral:


arctan x · ln2 x
Z
dx.
0 x2 − x + 1

Proposed by Vasile Mircea Popa, Sibiu, Romania

Solution 1 by Seán M. Stewart, King Abdullah University of Science and Technology, Saudi Arabia
Denote the integral to be evaluated by I. We show that

5π 4
I= √ .
81 3
1
We begin by enforcing a substitution of x 7→ x in the integral. This gives
∞ arctan( x1 ) log2 (x)
Z
I= dx.
0 1 − x + x2

Since
1
 π
arctan x = − arctan(x), x > 0,
2
allows the integral to be rewritten as

π ∞ log2 (x)
Z ∞
arctan(x) log2 (x) π ∞ log2 (x)
Z Z
I= dx − dx = dx − I,
2 0 x2 − x + 1 0 x2 − x + 1 2 0 x2 − x + 1
or

log2 (x)
Z
π
I= dx.
4 0 x2 − x + 1
Since
1 + x3 = (1 + x)(x2 − x + 1),
the above integral for I can be rewritten as

π ∞ (1 + x) log2 (x)
Z ∞ Z ∞
log2 (x) x log2 (x)
Z 
π
I= dx = dx + dx . (2)
4 0 1 + x3 4 0 1 + x3 0 1 + x3

To find the values of the two integrals appearing in (2), we consider the parametric integral
Z ∞ a−1
x
J(a, b) = dx, 0 < a < b.
0 1 + xb

Enforcing a substitution of u = xb produces


a

u b −1
Z
1
J(a, b) = du.
b 0 1+u

The integral that has appeared can be expressed in terms of a beta function. Here
1 a a 1 a  a π
J(a, b) = B ,1 − = Γ Γ 1− = .
b b b b b b b sin( aπ
b )

Here the connection between the beta and gamma function of

Γ(m)Γ(n)
B(m, n) = ,
Γ(m + n)

Mathematical Reflections 1 (2025) 25


has been used together with Euler’s reflexion formula for the gamma function of
π
Γ(x)Γ(1 − x) = .
sin(πx)
Thus
∂2J ∞
xa−1 log2 (x) π 3 (cos( 2aπ
b ) + 3)
Z
= dx = .
∂α2 0 1+x b 2b sin3 ( aπ
3
b )
Returning to the integral in (2) we see that
( )
π π 3 (cos( 2π
3 ) + 3) π 3 (cos( 4π
3 ) + 3) 5π 4
I = π4 J(1, 3) + J(2, 3) =

3 π + = √ ,
4 3
2 · 3 sin ( 3 ) 2 · 3 sin3 ( 2π
3
3 ) 81 3

as desired.

Solution 2 by Joshua Pité, Cambridge Rindge and Latin School, MA, USA
x ln2 x
The function f (x) = arctan
x2 −x+1
is continuous over (0, ∞).
Near 0 we have f (x) ∼ x(ln x) , and near ∞, f (x) = o x−3/2 . Hence the given integral converges. Denote
+ 2

R∞ x·ln2 x
I = 0 arctan
x2 −x+1
dx.
Using the substitution x = u1 we have

− arctan u1 (ln u)2 arctan u1 (ln u)2


Z 0  Z ∞ 
I= du = du.
∞ 1 − u + u2 0 u2 − u + 1

For u > 0, we know the identity arctan u1 = π2 − arctan u.




Thus  
Z ∞ 2 Z ∞ π − arctan u (ln u)2
arctan u (ln u) 2
I +I = 2−u+1
du + du
0 u 0 u2 − u + 1
 
Z ∞ arctan u + π − arctan u (ln u)2
π ∞ (ln u)2
Z
2
= du = du.
0 u2 − u + 1 2 0 u2 − u + 1
That is

(ln u)2
Z
π π
I= du =: J.
4 0 u2 − u + 1 4
R1 ln2 x
R∞ ln2 x
R1 2
Writing J = 0 x2 −x+1dx + 1 dx and u = 1/x again, we obtain that J = 2 0 x2ln−x+1
x2 −x+1
x
dx.
1 P n
We can write, for all x ∈ R, with |x| < 1, x2 −x+1 = n≥0 an x , where the coefficients an are

1
 n ≡ 0, 1 (mod 6)
an = −1 n ≡ 3, 4 (mod 6)

0 otherwise

so that
1
ln2 x 1
Z X Z
J =2 dx = 2 an xn ln2 x dx.
0 x2 − x + 1 0
n≥0

By integration by parts, with u = ln2 x, dv = xn dx, we get


Z 1 Z 1 n+1 Z 1
n 2 2 x −2
x ln x dx = − ln x dx = xn ln x dx
0 n+1 0 x n+1 0
With one more integration by parts,
Z 1
2
xn ln2 x dx = .
0 (n + 1)3

Mathematical Reflections 1 (2025) 26


" #
X 1 1 1 1
J =4 + − −
(6m + 1)3 (6m + 2)3 (6m + 4)3 (6m + 5)3
m≥0
" #
4 X 1 1 1 1
= 3 + − −
6 (m + 1/6)3 (m + 1/3)3 (m + 2/3)3 (m + 5/6)3
m≥0

Recall that the Hurwitz Zeta function is ζ(s, a) = ∞ 1


P
n=0 (n+a)s for ℜ(s) > 1 and a > 0.
We can then write
 
π
I = 3 ζ(3, 1/6) + ζ(3, 1/3) − ζ(3, 2/3) − ζ(3, 5/6) .
6
d n+1
The polygamma function is ψ (n) (x) = dx n+1 ln Γ(x), where Γ(x) is the Gamma function. We’ll use ψ
(2) (x),

which is related to ζ(3, x) by ψ (2) (x) = −2ζ(3, x).


We can use a reflection formula for the polygamma function:

d2
∀x ∈ (0, 1), ψ (2) (1 − x) − ψ (2) (x) = π cot(πx) = 2π 3 cot(πx) csc2 (πx).
dx2
5

Hence ψ (2) − ψ (2) 16 = 2π 3 cot π6 csc2 π
= 8 3π 3 .
   
6 √ 6
ψ (2) 4 (2) 2 = 8 3 π 3 .
 
Similarly 6 −ψ 6 9
After simplification, we conclude that


arctan x · ln2 x
Z
5 3 4
dx = π ≈ 3.47155.
0 x2 − x + 1 243

Solution 3 by Daniel Pascuas, Barcelona, Spain


The substitution y = 1/x yields that

arctan x · ln2 x ( π2 − arctan y) ln2 y

Z Z
dx = dy
0 x2 − x + 1 0 1 − y + y2
π ∞
Z ∞
ln2 x arctan x · ln2 x
Z
= dx − dx,
2 0 x2 − x + 1 0 x2 − x + 1
so
∞ ∞
arctan x · ln2 x ln2 x 5π 4
Z Z
π
2
dx = dx = √ ,
0 x −x+1 4 0 x2 − x + 1 81 3
because

ln2 x 20π 3
Z
dx = √ . (∗)
0 x2 − x + 1 81 3
We give two different proofs of (∗). The first one is just a standard evaluation of our integral by means of
the residues theorem. In the second proof we apply the monotone convergence theorem to write our integral
as the sum of a series which is evaluated by using the classical simple fraction expansion of the cotangent
(that is also a consequence of the residues theorem).
R∞ 2
Proof 1 of (∗). Note that 0 x2ln−x+1
x
dx = F ′′ (0), where F is the holomorphic function on the strip | Re z| < 1
defined by Z ∞
xz dx
F (z) = (| Re z| < 1).
0 x2 − x + 1
Now we are going to compute the integral F (z) by applying the residue theorem to the meromorphic function
z
wz
fz (w) = w2 −w+1 = (w+1)w
w3 +1
and the keyhole contour γδ,r,R , which we are going to define in a while. Here wz
denotes the branch of the z-power in the domain Ω = C \ [0, ∞) (where [0, ∞) = {x ∈ R : x ≥ 0}, as usual)

Mathematical Reflections 1 (2025) 27


π
whose value at w = −1 equals to eiπ . Thus fz is a meromorphic function on Ω whose only poles are e±i 3
(simple poles).
For − π2 < δ < π2 and 0 < r < R, let αδ,r,R be the line segment which goes from reiδ to Reiδ .
For 0 < δ < π2 and ϱ > 0, let βδ,ϱ be the arc of the circle centered at the origin with radius ϱ which goes
from ϱeiδ to ϱe−iδ counterclockwise. Moreover, if α is a path on the complex plane, then α−1 denotes the
path α traversed in the opposite direction. For 0 < δ < π2 and 0 < r < 1 < R, γδ,r,R is the concatenation of
−1 −1 π
αδ,r,R , βδ,R , α−δ,r,R , and βδ,r . Note that γδ,r,R is a closed rectifiable curve in Ω \ {e±i 3 }. Then the residue
theorem shows that
Z  
i π3 −i π3
 
fz (w) dw = 2πi Res fz (w); w = e + Res fz (w); w = e .
γδ,r,R

Since π π wz π
Res fz (w); w = ei 3 = lim π (w − ei 3 )fz (w) = lim π 1
π = √ e3
iz
and
i3 i 3 w − e−i 3 i 3
w→e w→e
π π wz 5π
Res fz (w); w = e−i 3 = lim π (w − e−i 3 )fz (w) = lim π i
1
π = − √ e 3
i 3
iz
,
−i 3 −i 3 w − e 3
w→e w→e

we have that Z
π 5π
iz iz 2π πiz − 2π 2π
fz (w) dw = 2π

3
(e 3 −e 3 )= √
3
e (e 3 iz −e 3
iz
).
γδ,r,R

Therefore
Z Z Z Z Z 
2π πiz − 2π 2π

3
e (e 3 iz −e 3
iz
)= fz (w) dw = + − − fz (w) dw.
γδ,r,R αδ,r,R βδ,R α−δ,r,R βδ,r

Then let δ ↘ 0 to get that


R 2π 2π
xz dx Rz eizθ iReiθ dθ rz eizθ ireiθ dθ
Z Z Z
2π πiz − 2π 2π
√ e (e 3 iz −e 3
iz
) = (1 − e 2πiz
) + − .
3
r x2 − x + 1 0 R2 e2iθ − Reiθ + 1 0 r2 e2iθ − reiθ + 1
But

Rz eizθ iReiθ dθ R1+Re z e2π| Re z|
Z
≤ 2π −→ 0, as R ↗ ∞, and
0 R2 e2iθ − Reiθ + 1 R2 − R − 1

rz eizθ ireiθ dθ r1+Re z e2π| Re z|
Z
≤ 2π −→ 0, as r ↘ 0, for | Re z| < 1.
0 r2 e2iθ − reiθ + 1 1 − r − r2
It follows that
2π 2π

xz dx 2π eiπz (e− 3 iz − e 3 iz
2π sin( 2π
3 z) 4π sinc( 2π
3 z)
Z
)
F (z) = = √ =√ = √ (| Re z| < 1),
0 x2 − x + 1 3 (1 − e2πiz ) 3 sin(πz) 3 3 sinc(πz)
where

sin z X (−1)k 2k
sinc z = = z (z ∈ C),
z (2k + 1)!
k=0
which is an entire function without zeros on the strip | Re z| < 1. As a consequence,

4π d2 sinc( 2π 4π d 2π sinc′ ( 2π z) sinc(πz) − π sinc( 2π z) sinc′ (πz)


   
′′ 3 z) 3 3 3
F (z) = √ = √ ,
3 3 dz 2 sinc(πz) 3 3 dz sinc2 (πz)
3 3
for −1 < Re z < 1, and therefore F ′′ (0) = − 27√ sinc′′ (0) =
20π 20π
√ , since sinc(0) = 1, sinc′ (0) = 0, and
3 81 3
sinc′′ (0) = − 31 . Hence we finally obtain (∗).
Proof 2 of (∗). The substitution y = 1/x yields that
Z ∞
ln2 x ln2 y ln2 x
Z 1 Z 1
dx = dy = dx,
1 x2 − x + 1 0 1−y+y
2 2
0 x −x+1

Mathematical Reflections 1 (2025) 28


so
∞ ∞
ln2 x 1
ln2 x 1
ln2 x 1
(1 + x) ln2 x
Z Z Z Z Z
dx = + dx = 2 dx = 2 dx.
0 x2 − x + 1 0 1 x2 − x + 1 0 x2 − x + 1 0 1 + x3

Since
∞ ∞ ∞
1 X
3 n
X
3 2n 3 2n+1
 X
= (−x ) = (x ) − (x ) = (1 − x3 )x6n (0 < x < 1),
1 + x3
n=0 n=0 n=0

the monotone convergence theorem shows that


∞ ∞
ln2 x 1
(1 + x) ln2 x 1
Z Z X Z
dx = 2 dx = 2 (1 + x)(1 − x3 )x6n ln2 x dx
0 x2 − x + 1 0 1 + x3
n=0 0

X 
=2 I6n + I6n+1 − I6n+3 − I6n+4 ,
n=0

where Z 1
Im = xm ln2 x dx (m = 0, 1, 2, . . . ).
0
We evaluate Im by integration by parts:
Z 1 h Z 1 
1 2 m+1 1 m+1 2
i1
m
Im = ln x dx = x ln x − 2 x ln x dx
m+1 0 m+1 0 0
Z 1 h i1 Z 1 
2 m+1 2 m+1 m 2
=− ln x dx =− x ln x − x dx = .
(m + 1)2 0 (m + 1)2 0 0 (m + 1)3

Therefore
∞ ∞
ln2 x
Z X 
dx = 2 I 6n + I 6n+1 − I 6n+3 − I 6n+4
0 x2 − x + 1
n=0
∞  
X 1 1 1 1
=4 + − −
(6n + 1)3 (6n + 2)3 (6n + 4)3 (6n + 5)3
n=0
∞  
X 1 1 1 1
=4 + + +
(6n + 1)3 (6n + 2)3 (2 − 6(n + 1))3 (1 − 6(n + 1))3
n=0
∞   ∞  
X 1 1 1 X 1 1
=4 + = + .
n=−∞
(6n + 1)3 (6n + 2)3 54 n=−∞ (n + 16 )3 (n + 13 )3

Now we evaluate this series by a nice complex variable argument. Recall the classical simple fraction
expansion of the cotangent:

X 1
π cot(πz) = (z ∈ C \ Z),
n=−∞
z+n

where the series converges uniformly on any compact subset of C \ Z. It follows that
∞ ∞
1 X d2 π d2
 
X 1 1 cos(πz)
= = cot(πz) = π 3 (z ∈ C \ Z).
n=−∞
(z + n)3 2
2 n=−∞ dz z + n 2 dz 2 sin3 (πx)

Hence we finally obtain (∗):



ln2 x π 3 cos( π6 ) cos( π3 ) 2π 3 √ 20π 3
Z    
1
dx = + = 3 + √ = √ .
0 x2 − x + 1 54 sin3 ( π6 ) sin3 ( π3 ) 27 3 3 81 3

Mathematical Reflections 1 (2025) 29


Editor’s Comment. In order to calculate the integral

π ∞ ln2 u
Z
I= du,
4 0 u2 − u + 1

Henry Ricardo concluded immediately by using Formula 4.261.2 in Table of Integrals, Series, and Products
(Sixth Edition) by I. S. Gradshteyn and I. M. Ryzhik, which states that

ln2 u 20π 3
Z
du = √ ,
0 u2 − u + 1 81 3

giving I = 5π 4 /81 3.

Also solved by Brian Bradie, Christopher Newport University, Newport News, VA, USA; Devis Alvarado,
UNAH y UPNFM, Tegucigalpa, Honduras; G. C. Greubel, Newport News, VA, USA; Henry Ricardo, Westch-
ester Area Math Circle, New York, USA; Nicuşor Zlota, “Traian Vuia” Technical College, Focşani, Romania;
Paolo Perfetti, Università degli studi di Tor Vergata Roma, Italy; Theo Koupelis, Clark College, Washington,
USA; Prasanna Pawar, AMMOC, Bengaluru, India; Srijan Sundar, Oxford, UK.

Mathematical Reflections 1 (2025) 30


U690. Find all the complex numbers z such that

(Γ(z + 1))3 + (z − 1)3 (Γ(z − 1))3 = (z + 1)3 + 3 (Γ(z))3 ,




where Γ(z) is the Gamma function.

Proposed by Alessandro Ventullo, Milan, Italy

Solution 1 by the author


The given equality can be rewritten as

(Γ(z + 1))3 − (z + 1)3 (Γ(z))3 + (z − 1)3 (Γ(z − 1))3 = 3 (Γ(z))3 .

If we set a = Γ(z + 1), b = −(z + 1)Γ(z) and c = (z − 1)Γ(z − 1), the given equality becomes

a3 + b3 + c3 = 3(Γ(z))3 .

Now, observe that for every complex number z we have Γ(z + 1) = zΓ(z), so

a + b + c = 0,

and from the identity a3 + b3 + c3 − 3abc = (a + b + c)(a2 + b2 + c2 − ab − bc − ca), we get

3abc = 3 (Γ(z))3 ,

i.e.
−(z 2 − 1)Γ(z + 1)Γ(z)Γ(z − 1) = (Γ(z))3 .
By using once again Γ(z + 1) = zΓ(z), we get

−(z 2 − 1)z (Γ(z))2 Γ(z − 1) = (Γ(z))3

and since Γ(z) ̸= 0 for all complex numbers z, then

−(z 2 − 1)zΓ(z − 1) = Γ(z),

i.e.
−(z + 1)z = 1.

2 −1 ± i 3
Therefore, z + z + 1 = 0, which gives z = .
2
Solution 2 by Brian Bradie, Christopher Newport University, Newport News, VA, USA
Because
Γ(z + 1) = zΓ(z) and (z − 1)Γ(z − 1) = Γ(z),
the given equation is equivalent to

z 3 (Γ(z))3 + (Γ(z))3 = ((z + 1)3 + 3)(Γ(z))3 ,

i.e.
3(Γ(z))3 (z 2 + z + 1) = 0.

1 3
Since there is no z ∈ C for which Γ(z) = 0, it follows that z2 + z + 1 = 0, i.e. z = − ± i .
2 2
Also solved by G. C. Greubel, Newport News, VA, USA; Nicuşor Zlota, “Traian Vuia” Technical College,
Focşani, Romania; Paolo Perfetti, Università degli studi di Tor Vergata Roma, Italy; Seán M. Stewart, King
Abdullah University of Science and Technology, Saudi Arabia; Theo Koupelis, Clark College, Washington,
USA; Daniel Pascuas, Barcelona, Spain; Diar Gashi, University of Primorska, Slovenia; Srijan Sundar,
Oxford, UK.

Mathematical Reflections 1 (2025) 31


Olympiad problems

O685. Let a, b, c be positive real numbers such that a + b + c = 1. Prove that


ab bc ca
+ + + 3(ab + bc + ca) ≥ 2.
c a b

Proposed by Adrian Andreescu, University of Texas at Dallas, USA

Solution 1 by Srijan Sundar, Oxford, UK


By the AM-GM Inequality we have
  X b c
ab bc ca
(a + b + c) + + = ab + bc + ca + a2 +
c a b cyc
c b
≥ ab + bc + ca + 2(a2 + b2 + c2 ).

Hence, since a + b + c = 1, we get


ab bc ca
+ + + 3(ab + bc + ca) ≥ 2(a + b + c)2 = 2.
c a b
1
The equality holds if and only if a = b = c = .
3
Solution 2 by Theo Koupelis, Clark College, Washington, USA
The desired inequality is equivalent to

a2 b2 + b2 c2 + c2 a2 + 3abc(ab + bc + ca) ≥ 2abc.

Homogenizing the desired inequality, it suffices to show that

(a + b + c)(a2 b2 + b2 c2 + c2 a2 ) + 3abc(ab + bc + ca) ≥ 2abc(a + b + c)2 .

Expanding and simplifying we get

a3 (b − c)2 + b3 (c − a)2 + c3 (a − b)2 ≥ 0,


1
which is obvious. The equality occurs when a = b = c = .
3
Also solved by Hoang-Bao Le, Dublin, Ireland; Marin Chirciu, Colegiul National Zinca Golescu Piteşti,
Romania; Nicuşor Zlota, “Traian Vuia” Technical College, Focşani, Romania; Paolo Perfetti, Università
degli studi di Tor Vergata Roma, Italy; Sundaresh H. R., Shivamogga, Karnataka, India; Anderson Torres,
Sao Paulo, Brazil; Daniel Pascuas, Barcelona, Spain; Nguyen Thanh Nam, Tu Son Secondary School,
Vietnam.

Mathematical Reflections 1 (2025) 32


O686. Let a, b, c, d be positive real numbers such that
X X
2 a+ ab + 3 = abcd.
cyc sym

Prove that √ √ √ √ p
4 4 4 4 4
bcd + cda + dab + abc ≤ 27(1 + a)(1 + b)(1 + c)(1 + d)

Proposed by An Zhenping, Xianyang Normal University, China

Solution by Theo Koupelis, Clark College, Washington, USA


The given condition is equivalent to
1 1 1 1
+ + + = 1.
a+1 b+1 c+1 d+1
 
1 1 1 1
Let (x, y, z, w) = , , , , where x, y, z, w ∈ (0, 1) and x + y + z + w = 1. Thus, the
a+1 b+1 c+1 d+1
desired inequality is equivalent to
s r
X 4 (1 − y)(1 − z)(1 − w) 27
≤ 4 ,
yzw xyzw
or Xp
4
3x(1 − y)(1 − z)(1 − w) ≤ 3.
Using AM-GM we get
p
2 + 4x = 3x + (1 − y) + (1 − z) + (1 − w) ≥ 4 4 3x(1 − y)(1 − z)(1 − w)

with similar expressions for 2 + 4y, 2 + 4z, 2 + 4z. Adding these expressions we get
Xp
4 8 + 4(x + y + z + w)
3x(1 − y)(1 − z)(1 − w) ≤ = 3,
4
which completes the proof.

Also solved by Nicuşor Zlota, “Traian Vuia” Technical College, Focşani, Romania.

Mathematical Reflections 1 (2025) 33


O687. Let x, y, z be real numbers such that xy + yz + zx = 1 and x + y + z > 0. Prove that
√ √ p
xy z 2 + 1 + yz x2 + 1 + zx y 2 + 1 2
≤ .
x+y+z 3
Proposed by Marius Stănean, Zalău, Romania

Solution 1 by the author


The inequality can be rewritten as
p p p
xy (y + z)(z + x) + yz (x + y)(z + x) + zx (y + z)(x + y) 2
≤ .
(x + y + z)(xy + yz + zx) 3
We have two cases: x + y, y + z, z + x ≤ 0 or x + y, y + z, z + x ≥ 0. In first case with the substitutions x →
−x, y → −y, z → −z, we are in the second case. Hence, we will prove the inequality for x+y, y +z, z +x ≥ 0.
It is obvious that only one of the numbers x, y, z can be negative. Also, we have
√ √
q p
x + z + y + z = x + z + y + z + 2 (x + z)(y + z)
q p
= x + y + 2z + 2 z 2 + 1

> x + y.
√ √ √
Therefore, there is a triangle ABC with sides a = y + z, b = z + x, c = x + y. The inequality becomes
X X
3 ab(b2 + c2 − a2 )(c2 + a2 − b2 ) ≤ (a2 + b2 + c2 ) (b2 + c2 − a2 )(c2 + a2 − b2 ),
cyc cyc
or !
X X X X X X X
3 3 3 4 2 2 2 4
6 a b + 6abc a −3 ab a ≤ a 2 a b − a ,
cyc cyc cyc cyc cyc cyc cyc
or ! !
X X X X X X X X
3 3 3 2 2 2 2 2 4
6 a b + 6abc a −6 ab a b ≤ a −3 ab 2 a b − a ,
cyc cyc cyc cyc cyc cyc cyc cyc
or ! ! !
X X X X X X X
6abc a 2 ab − a2 + a2 − 3 ab 2 a2 b2 − a4 − 36a2 b2 c2 ≥ 0.
cyc cyc cyc cyc cyc cyc cyc
Using the known identities in a triangle
abc = 4Rrs, ab + bc + ca = s2 + r2 + 4Rr, a2 + b2 + c2 = 2(s2 − r2 − 4Rr),
after performing the calculations, it becomes
48Rrs2 (4r2 + 16Rr) − 16r2 s2 (s2 + 5r2 + 20Rr) − 36(16R2 r2 s2 ) ≥ 0,
or
s2 ≤ 12R2 − 8Rr − 5r2 .
Since by Gerretsen’s Inequality, we have
s2 ≤ 4R2 + 4Rr + 3r2 ,
it suffices to show that
4R2 + 4Rr + 3r2 ≤ 12R2 − 8Rr − 5r2 .
This factors as
(R − 2r)(2R + r) ≥ 0,
which is true by Euler’s Inequality. The equality holds when x = y = z.

Also solved by Arkady Alt, San Jose, California, USA; Marin Chirciu, Colegiul National Zinca Golescu
Piteşti, Romania; Nicuşor Zlota, “Traian Vuia” Technical College, Focşani, Romania; Theo Koupelis, Clark
College, Washington, USA; Srijan Sundar, Oxford, UK.

Mathematical Reflections 1 (2025) 34


O688. Inside the square ABCD the points E, F are taken such that∡F DE = ∡EBF = 45◦ It is given that
area F CD+area DEA+area EF B = a2 . Find the length of the side AB.

Proposed by Mihaela Berindeanu, Bucharest, Romania

Solution by Kousik Sett, India


Let lines parallel to AD, passing through E, F meet AB, DC at G, I and H,J, respectively. Similarly, let
lines parallel to AB, passing through E, F meet AD, BC at K, M and L, N , respectively.
Let P = IJ ∩ KL, Q = GH ∩ M N , and let x be the side length of the square ABCD such that x =
h1 + h2 + h3 = k1 + k2 + k3 , where h1 = HQ, h2 = QE, h3 = EG, and k1 = M Q, k2 = QF , k3 = F N ,
respectively.
Assume that ∠ABE = α, ∠CBF = β, ∠CDF = γ, and ∠ADE = δ. Since ∠F DE = ∠EBF = 45◦ , we
infer that α + β = 45◦ and γ + δ = 45◦ .
Lines GH, IJ, KL, and M N divide the square ABCD into nine rectangles as shown at the figure below.
Let [XY Z · · · W ] denotes the area of the polygon [XY Z · · · W ].

From right triangles BGE and BN F , we get


h3 k3
tan α = and tan β = .
x − k1 x − h1
Since α + β = 45◦ , we have
tan α + tan β + tan α · tan β = 1.
Substituting the expressions for tan α, tan β, and simplifying, we obtain

x2 − x(h1 + k1 + h3 + k3 )x + h1 h3 + k1 k3 + h1 k1 − h3 k3 = 0. (3)

From right triangles DJF and DKE, we get


h1 k1
tan γ = and tan δ = .
x − k3 x − h3
Since γ + δ = 45◦ , we have
tan γ + tan δ + tan γ · tan δ = 1.
Substituting the expressions for tan γ, tan δ, and simplifying, we obtain

x2 − x(h1 + k1 + h3 + k3 )x + h1 h3 + k1 k3 − h1 k1 + h3 k3 = 0. (4)

Mathematical Reflections 1 (2025) 35


From (3) and (4), we obtain

h1 k1 = h3 k3 , i.e., [M QHD] = [IBLP ]. (5)

Observe that

[F CD] + [DEA] + [EF B]


1
= ([M N CD] + [AGHD]) + [GBN Q] − [GBE] − [BN F ] − [EF Q]
2
1 1
= ([M N CD] + [AGHD]) + [GBN Q] − ([GBLE] + [IBN F ] + [EP F Q])
2 2
1 1
= ([M N CD] + [AGHD]) + [GBN Q] − ([GBN Q] + [IBLP ])
2 2
1 1
= ([M QHD] + [QN CH] + [M QHD] + [AGQM ]) + ([GBN Q] − [IBLP ])
2 2
1 1
= ([M QHD] + [QN CH] + [AGQM ] + [GBN Q]) + ([M QHD] − [IBLP ])
2 2
1
= ([ABCD]). [by (5)]
2

Since [F CD] + [DEA] + [EF B] = a2 , we 2 2 2
√ have [ABCD] = 2a , i.e., x = 2a . So, x = 2a.
Therefore, the length of the side AB = 2a.

Also solved by Theo Koupelis, Clark College, Washington, USA.

Mathematical Reflections 1 (2025) 36


5
O689. Prove that is the minimum positive value of the constant k such that
2
r r r r
b+c c+a a+b 2
+ + ≥3
ka + b + c kb + c + a kc + a + b k+2
holds for any nonnegative real numbers a, b, c with a + b + c > 0.

Proposed by Vasile Cı̂rtoaje, Oil-Gas University, Ploieşti, Romania

Solution 1 by the author r


2 5 5
For b = c = 0, the inequality becomes 2 ≥ 3 , which is equivalent to k ≥ . To show that is the
k+2 2 2
minimum value of the constant k, we need to prove the inequality

r r r
b+c c+a a+b
+ + ≥ 2.
5a + 2b + 2c 5b + 2c + 2a 5c + 2a + 2b
Let
p = a + b + c, q = ab + bc + ca, r = abc.
Applying the Cauchy-Schwarz inequality twice, we have
[ (b + c)]2
r
4(a + b + c)2
P
X b+c
≥P √ ≥ pP
5a + 2b + 2c (b + c)3/2 5a + 2b + 2c [ (b + c)] [ (b + c)2 (5a + b + c)]
P
s
4p2 8p3
=p = 3
.
4p − pq + 9r
P 3 P
2p [4 a + 11 ab(a + b) + 30abc]
So, it suffices to show that
8p3
≥ 2,
4p3 − pq + 9r
which is equivalent to the known inequality pq ≥ 9r.
The proof is completed. The equality occurs for a = b = c, and also for a = b = 0 or b = c = 0 or c = a = 0.

Solution 2 by Paolo Perfetti, Università degli studi di Tor Vergata Roma, Italy
Let b = c = 0. The inequality becomes
r
2
2≥3 ⇐⇒ k ≥ 5/2
k+2
By taking k = 5/2 and using Hölder’s inequality we get
r r !3
X 2(b + c) X 2(b + c) X X
(2b + 2c)2 (5a + 2b + 2c) ≥ (2b + 2c)
cyc
5a + 2b + 2c cyc
5a + 2b + 2c cyc cyc

The inequality is equivalent to


!3
X X 2
(2b + 2c) ≥ (2b + 2c)2 (5a + 2b + 2c) · 9 5
cyc cyc 2 +2

Upon simplifying this last inequality is


a2 b + ab2 + b2 c + bc2 + c2 a + ca2 ≥ 6abc
which is evidently true by the AM-GM Inequality. The equality occurs when a = b = c or when a = b = 0
or b = c = 0 or c = a = 0.

Also solved by Nicuşor Zlota, “Traian Vuia” Technical College, Focşani, Romania; Theo Koupelis, Clark
College, Washington, USA.

Mathematical Reflections 1 (2025) 37


O690. Prove that there is an infinite sequence S of primes such that for any finite subset A ⊂ S the sum of
elements of A is not a perfect power.

Proposed by Mohammadreza Ghelichkhani, Tehran, Iran

Solution by the author


We construct S inductively. First, we add 2 to S. Now, assume that at some stage S = {p1 , p2 , . . . , pt }, and
let the set of all possible sums of elements from its subsets be A = {a1 , a2 , . . . , an }, where n = 2t .
Let qi be the i-th prime number that does not divide any element in A. We choose the next prime number,
pt+1 , such that for each 1 ≤ i ≤ n,
pt+1 + ai ≡ qi (mod qi2 ).
It is clear that the solution to this congruence is coprime to qi for each i. Therefore, by Dirichlet’s theorem,
there exists such a prime number.

Also solved by Juan José Granier, Cencosud, Chile.

Mathematical Reflections 1 (2025) 38

You might also like